Stars that have more mass than the sun will end their lives as neutron stars.
Which stars will die and become neutron stars?If a star has a mass of 1.5 to 2 or 3 times that of the Sun, it will eventually collapse even more, forming a neutron star with a possible diameter of 20 kilometers. In neutron stars, the gravitational force has overcome the electrons' ability to resist compression and compelled them to combine with protons to form neutrons.
After its formation, a neutron star produces neither light nor heat on its own. From an initial temperature of 600,000 degrees Kelvin (1 million degrees Fahrenheit), its latent heat will gradually decrease over millions of years, putting an end to its life as the icy, dead remnant of a once-brilliant star.
To know more about mass visit:
https://brainly.com/question/29515102
#SPJ4
17.5 l of h2 is stored at a pressure of 34.7 kpa. what volume would the gas take up at stp? (assume constant temperature)
The volume of H2 gas at STP would be approximately 6.0 L.
To determine the volume of H2 gas at STP when 17.5 L of H2 is stored at a pressure of 34.7 kPa, you can use the Boyle's Law formula, which states that P1V1 = P2V2 for a constant temperature.
the given values.
P1 = 34.7 kPa (Initial pressure)
V1 = 17.5 L (Initial volume)
P2 = 101.3 kPa (Pressure at STP)
Solve for V2 (the volume at STP) using the Boyle's Law formula:
P1V1 = P2V2
Rearrange the formula to find V2:
V2 = (P1V1) / P2
Plug in the values and calculate V2:
V2 = (34.7 kPa * 17.5 L) / 101.3 kPa
Calculate the result:
V2 ≈ 6.0 L
So, the volume of H2 gas at STP would be approximately 6.0 L.
Learn more about Boyle's Law
brainly.com/question/21184611
#SPJ11
Two narrow slits are illuminated by a laser with a wavelength of 520 nm. The interference pattern on a screen located 5.50 m away shows that the second-order bright fringe is located y = 8.40 cm away from the central bright fringe. Calculate the distance between the two slits. Submit Answer Tries 0/99 The screen is now moved 1.0 m further away. What is the new distance between the central and the second-order bright fringe? Submit Awwww Tries 0/99
the new distance between the central and the second-order bright fringe, when the screen is moved 1.0 m further away, is approximately 9.92 m.
To solve this problem, we can use the equation for the location of bright fringes in the double-slit interference pattern:
y = (λL) / d
where:
y is the distance from the central bright fringe to the desired fringe (in this case, the second-order bright fringe),
λ is the wavelength of light,
L is the distance between the slits and the screen,
and d is the distance between the two slits.
Let's first calculate the distance between the slits using the given information:
λ = 520 nm
= 520 × \(10^{(-9)}\) m
L = 5.50 m
y = 8.40 cm
= 8.40 × \(10^{(-2) }\)m
Using the equation, we can rearrange it to solve for d:
d = (λL) / y
Substituting the values, we have:
d = (520 ×\(10^{(-9)}\) m × 5.50 m) / (8.40 × \(10^{(-2)}\) m)
d ≈ 3.40 ×\(10^{(-3)}\) m
So, the distance between the two slits is approximately 3.40 × 10^(-3) m.
Now, let's calculate the new distance between the central and the second-order bright fringe when the screen is moved 1.0 m further away: L' = L + 1.0 m = 5.50 m + 1.0 m = 6.50 m
Using the same equation, we can find the new distance:
y' = (λL') / d
Substituting the values:
y' = (520 × 10^(-9) m × 6.50 m) / (3.40 × 10^(-3) m)
y' ≈ 9.92 m
To know more about fringe visit:
brainly.com/question/31387359
#SPJ11
Two friends are carrying a 200-kg crate up a flight of stairs. The crate is 1.25 m long and 0.500 m high, and its center of gravity is at its center. The stairs make a 45.0° angle with respect to the floor. The crate also is carried at a 45.0° angle, so that its bottom side is parallel to the slope of the stairs (Fig. 2). If the force each person applies is vertical, what is the magnitude of each of these forces?
Let consider the whole system to be at equilibrium where the net force is zero. The forces \(F_A\) and \(F_B\) are acting in the upward direction while the weight (w) of the crate is acting in the downward direction.
The force at equilibrium in the y-direction is computed as:
\(\sum F_y = ma_y\)
\(F_A +F_B= mg\)
\(\mathbf{F_A = mg -F_B}\) ---- (1)
Recall that the weight w = mg
where;
mass (m) = 200 g g(constant gravity) = 9.8 m/s²w = 200 × 9.8
w = 1960 N
The torque about the bottom end is also zero. i.e.
\(\sum \tau=0\)
∴
\(F_B L_Z - wL_w = 0\) --- (2)
\(F_B L_Z= wL_w\)
\(F_B = \omega (\dfrac{L_w}{L_z})\)
\(F_B =1960\times (\dfrac{0.375\times cos 45^0}{1.25\times cos 45^0})\)
\(F_B =1960\times (0.3)\)
\(\mathsf{F_B = 588 \ N}\)
Replacing the value for the force of B into equation (1); we have:
\(F_A = mg - F_B\)
\(F_A = 1960 - 588\)
\(\mathsf{F_A = 1372 N}\)
Thus, we can conclude that the magnitude of these forces i.e force B and force A are 588 N and 1372 N respectively.
Learn more about the equilibrium of forces here:
https://brainly.com/question/24018969?referrer=searchResults
Calculate the total power of 5 fans each of them draws a current of 0.8A at a potential difference of 220V
Given: No. of fans = 5
Current (I)= 0.8A
Voltage (V) 220V
To find: Total power P
Solution: We know that P=VI
Therefore for one fan P= 0.8*220
= 176 W
Total Power for 5 fans = 176*5
= 880W
NO LINKS OR UNUSEFUL ANSWERS
What is the trade off between the input and output forces when someone uses a screw?
the flipper hits the ball with a force of 50 N. if the pinball, mass of 0.2 kg, moves across the frictionless surface, with what acceleration does the pinball move after the force is applied
The pinball will move with an acceleration of 250 m/s² across the frictionless surface after the force is applied by the flipper.
How to determine the acceleration of the pinballTo find the acceleration of the pinball, we need to use Newton's Second Law of Motion, which states that force is equal to mass times acceleration (F=ma).
In this case, the force applied by the flipper is 50 N and the mass of the pinball is 0.2 kg.
Therefore, we can write the equation as 50 N = 0.2 kg x a.
To solve for the acceleration, we can divide both sides by the mass, which gives us a = 50 N / 0.2 kg.
Simplifying this equation, we get a = 250 m/s².
Learn more about Acceleration at
https://brainly.com/question/25876659
#SPJ11
HELP PLEASE ASAP! Analyze the diagram below. Each segment of the umbrella appears to be a different color. Explain why each section appears to be the color that you see.
Answer using 3 to 5 complete sentences.
Color perception is a complex process influenced by various factors such as the interaction between light and objects, the human visual system, lighting conditions, cultural influences, and personal experiences. Understanding these factors helps us appreciate the subjective nature of color perception and how it can vary among individuals and in different contexts.
Color perception is the process by which our brain interprets the electromagnetic radiation reflected or emitted by objects and translates it into the sensation of color. While the physical properties of an object, such as its surface reflectance and light absorption, play a role in color perception, it is also influenced by various subjective factors.
One important factor is the interaction between the light source and the object. The color of an object depends on the wavelengths of light it reflects or transmits. For example, an object that reflects only red light will appear red when illuminated with white light. Similarly, if an object absorbs all wavelengths except blue, it will appear blue under white light.
Another factor is the human visual system, which consists of specialized cells called cones that are sensitive to different ranges of wavelengths. The three types of cones respond primarily to short (blue), medium (green), and long (red) wavelengths. The combination and intensity of signals from these cones determine the perceived color. For instance, when both the red and green cones are stimulated, we perceive the color yellow.
Additionally, factors like lighting conditions, background colors, and individual differences in color perception can influence how we perceive an object's color. For example, the same object may appear differently under natural daylight compared to artificial lighting.
Furthermore, cultural and contextual factors can influence color perception. Different cultures may associate certain colors with specific meanings or emotions. Additionally, factors like memory, attention, and personal experiences can influence color perception. For instance, an object may trigger different associations or emotional responses based on an individual's past experiences with similar objects.
For more such information on: Color perception
https://brainly.com/question/24871034
#SPJ8
The question probable may be:
Explain the concept of color perception and the factors that influence the perceived color of an object. Provide examples to support your explanation.
Select the correct answer. What happens to sound waves after they are received by our ears?
A. The longitudinal compression waves are transmitted directly to the brain.
B. The vibrational energy of the sound waves is converted into a mix of electric and mechanical signals and sent to the brain through two separate pathways.
C. The vibrational energy of the waves is converted into electrical signals and sent to the brain.
D. The longitudinal compression waves are converted into electromagnetic waves and sent to the brain.
Answer:
C
Explanation:
Answer:
C on plato
Explanation:
the manufacturer of a 12 v car headlight specifies it will draw a current of 6 a. you would like to check this claim with an ammeter designed to measure currents up to 10 a and having a resistance of 0.1 ohms. which of the two circuits in the attached figure represents a circuit where the ammeter correctly measures the current in the headlight?
Since an ammeter is meant to measure current and is a low resistance device, it is always connected in series with the circuit element through which current is to be measured.
What is electric current?Current of a system is defined as the rate at which the electrons flow from a given point in the absolute electrical circuit.
a. Since, Current in two or more components connected in series is always the same. So circuit (a) will be used.
b. As per manufacturer's claim, resistance of bulb r = V /I.
= 12 /6 = 2 Ω.
Total resistance in circuit a, R = 2 +0.1 = 2.1 A
Ammeter reading I' = V /R
= 12 /2.1 = 5.71 A
Which is close enough to claimed value of 6 A.
c. Now the bulb and ammeter are connected in parallel, so the voltage across both will be the same.
Current through ammeter I' = V /Ra = 12 /0.1 = 120 A
Which is 20 times greater than claimed value.
Learn more about electric current, here:
https://brainly.com/question/29766827
#SPJ1
The question is incomplete, but most probably the complete question is,
The manufacturer of a 12V car headlight specifies it will draw a current of 6A. You would like to check this claim with an ammeter designed to measure currents up to 10Aand having a resistance of 0.1 Ohms.
a)Which of the two figures represents a circuit where the ammeter correctly measures the current in the headlight?
b) How much current (in A) would flow in the ammeter for Circuit a?
c) How much current (in A) flows through the ammeter for Circuit b?
What happens to the electric force between two particles if the distance
between them is doubled?
A. It decreases to half the original force.
B. It increases to twice the original force.
C. It increases to 4 times the original force.
D. It decreases to 1/4 of the original force.
Answer:
D.
Explanation:
Force and distance are inversely related, so a greater distance results in a weaker force.
How would you feel if fertility fraud was committed against you? Think about individuals that go through infertility and their desire to become a parent
Fertility fraud is a deceptive act that involves medical professionals using their own sperm or someone else's without the patient's consent during fertility treatments.
It is a violation of trust and a grave breach of ethical standards that can have significant emotional and psychological consequences for those who experience it.
Individuals and couples who undergo fertility treatments often do so with the hope and expectation of having a child. Fertility fraud can shatter that hope, leaving individuals feeling violated, betrayed, and devastated. The emotional pain and trauma that follow can be long-lasting, and can impact their personal relationships and mental health.
The desire to become a parent is a fundamental human need and is often accompanied by feelings of loss, grief, and despair when fertility treatments fail. The emotional burden of fertility fraud compounds these feelings, leading to an indescribable sense of betrayal and a loss of trust in the medical system.
It is important to acknowledge and support those who have experienced fertility fraud, and to advocate for legal and ethical standards that prioritize patient autonomy, informed consent, and transparency in all aspects of fertility treatments
Know more about violation here:
https://brainly.com/question/1274113
#SPJ11
n which order did the events forming our solar system occur?
The solar nebula became hot and dense pulling in more gas.This flattened into a rotating disk. It spun faster and faster, forming the Sun.
Gas was pulled toward the center, forming the Sun. Gas flattened into a rotating disk and became hot and dense, forming a solar nebula that spun faster and faster.
Gas flattened into a rotating disk and became hot and dense, forming a solar nebula that spun faster and faster. Gas was pulled toward the center, forming the Sun.
The solar nebula spun faster and faster and flattened into a rotating disk. Most of the gas was pulled toward the center, where it became hot and dense, forming the Sun.
Answer:
The solar nebula became hot and dense because of that it pulling in more gas. This flattened into a rotating disk. It spun faster and faster, forming the Sun.
Explanation:
hope this helps
The solar nebula became hot and dense because of that it pulling in more gas. This flattened into a rotating disk. It spun faster and faster, forming the Sun. This order did the events forming our solar system occur.
What is Solar nebula ?In the so-called nebular hypothesis of the genesis of the solar system, the Sun and planets originated by condensation from a gaseous cloud. In 1734, Swedish philosopher Emanuel Swedenborg claimed that the planets arose from a nebular crust that enveloped the Sun before breaking apart. Immanuel Kant, a German philosopher, proposed in 1755 that the Sun and planets were created by a slow rotating nebula that was eventually pushed together by its own gravitational force and flattened into a spinning disc. In 1796, the French astronomer and mathematician Pierre-Simon Laplace presented a similar concept, but with the planets forming before the Sun. The Kant-Laplace theories were criticised by the British physicist James Clerk in the late nineteenth century.
To know more about solar Nebula :
https://brainly.com/question/28715616
#SPJ5.
A copper rod is sliding on two conducting rails that make an angle of 19 degree with respect to each other, as in the drawing. The rod is moving to the right with a constant speed of 0.60 m/s. A 0.35-T uniform magnetic field is perpendicular to the plane of the paper. Determine the magnitude of the average emf induced in the triangle ABC during the 6.0-s period after the rod has passed
The magnitude of the average emf induced in the triangle ABC during the 6.0-s period after the rod has passed is E m f = 0.124 V.
Calculation:-
As B is constant, then
E m f = B (A f - Ai) / t
Here,
Ai = 0
A f = b h /2
As
b = 0.60 m/s * 6.0 s = 3.6 m
h = b tan 19 = 0.5457 m
A f = 1.9646 m^2
e m f = 0.124 V
A magnetic field is a vector area that describes the magnetic impact on moving electric costs, electric powered currents, and magnetic substances. A moving price in a magnetic area reviews a pressure perpendicular to its very own pace and to the magnetic area.
Learn more about magnetic field here:-
https://brainly.com/question/7802337
#SPJ4
if the relative density of a liquid is 0.34 what is the density of the liquid
Answer:
340 kg/m³
Explanation:
\(Relative density \: = \frac{density \: ofsubstance}{density \: of \: equal \: volume \: ofwater} \)
density of water = 1000 kg/m³
density of the liquid = 0.34 × 1000
= 340 kg/m³
What is a carrier wave?
O a wave used to heat up and cook food
a wave that transmits speech, music, and other signals
a wave used to heat up and cook food
O a wave that reflects off bone in an x-ray
Answer:
A wave that transmits speech, music, and other signals
Explanation:
Hope it helps
(Give a thank OR mark as brainliest if it's the right answer)
Answer:
a wave that transmits speech, music, and other signals :)
Explanation:
Vector A has a magnitude of 8.0 m and points 30 degrees north of east; vector B has a magnitude of 6.0 m and points 30 degrees west of north; and vector C has a magnitude of 5.0 m and points 30 degrees west of south. The resultant vector A + B + C is given by:
A) 2.1 m at an angle 66 degrees east of north.
B) 5.9 m at an angle 74 degrees north of east.
C) 2.7 m at an angle 74 degrees north of east.
D) 5.1 m at an angle 74 degrees north of east.
E) 4.8 m at an angle 74 degrees east of north.
The resultant vector A + B + C is approximately 5.1 m at an angle 74 degrees north of east.
To find the resultant vector A + B + C, we need to break down each vector into its horizontal (x) and vertical (y) components, and then add them together.
Given:
Vector A: magnitude = 8.0 m, angle = 30 degrees north of east
Vector B: magnitude = 6.0 m, angle = 30 degrees west of north
Vector C: magnitude = 5.0 m, angle = 30 degrees west of south
Let's calculate the x and y components for each vector:
For Vector A:
A_x = 8.0 m × cos(30°)
A_x = 8.0 m × 0.866
A_x ≈ 6.928 m
A_y = 8.0 m × sin(30°)
A_y = 8.0 m ×0.5
A_y = 4.0 m
For Vector B:
B_x = 6.0 m × sin(30°)
B_x = 6.0 m × 0.5
B_x = 3.0 m
B_y = 6.0 m × cos(30°)
B_y = 6.0 m × 0.866
B_y ≈ 5.196 m
For Vector C:
C_x = 5.0 m × sin(30°)
C_x = 5.0 m ×0.5
C_x = 2.5 m
C_y = 5.0 m × cos(30°)
C_y = 5.0 m × 0.866
C_y ≈ 4.33 m
Now, let's add up the x and y components:
Resultant x component = A_x + B_x + C_x
Resultant x component = 6.928 m + 3.0 m + 2.5 m
Resultant x component ≈ 12.428 m
Resultant y component = A_y + B_y + C_y
Resultant y component = 4.0 m + 5.196 m + 4.33 m
Resultant y component ≈ 13.526 m
Finally, we can find the magnitude and angle of the resultant vector:
Resultant magnitude = sqrt((Resultant x component)² + (Resultant y component)²)
Resultant magnitude = sqrt((12.428 m)^2 + (13.526 m)^2)
Resultant magnitude ≈ 18.015 m
Resultant angle = arctan(Resultant y component / Resultant x component)
Resultant angle = arctan(13.526 m / 12.428 m)
Resultant angle ≈ 48.413°
Considering the answer choices provided:
A) 2.1 m at an angle 66 degrees east of north.
B) 5.9 m at an angle 74 degrees north of east.
C) 2.7 m at an angle 74 degrees north of east.
D) 5.1 m at an angle 74 degrees north of east.
E) 4.8 m at an angle 74 degrees east of north.
The closest match to our calculated result is:
D) 5.1 m at an angle 74 degrees north of east.
Therefore, the resultant vector A + B + C is approximately 5.1 m at an angle 74 degrees north of east.
To know more about resultant vector
https://brainly.com/question/28188107
#SPJ2
-.The Scablands in Washington are large
areas of erosion due to immense flooding. A
student claims the flooding of the Scablands
is due to erosion and melting of glaciers
which increased the amount of water until
it overwhelmed the dams. Using the claim
and map shown, create a claim-evidence-
reasoning model.
Water flo
Answer:
Here is a claim-evidence-reasoning (CER) model for the student's claim about the flooding of the Scablands:
Claim: The flooding of the Scablands is due to erosion and melting of glaciers which increased the amount of water until it overwhelmed the dams.
Evidence:
Erosion patterns in the Scablands that suggest a large amount of water was present in the region.
The presence of glacial deposits in the area.
The location of the Scablands near glacial formations that would have melted and contributed to the increase in water.
Reasoning: The evidence supports the idea that melting glaciers and erosion caused a significant increase in water in the region, which then overwhelmed the dams and caused flooding in the Scablands. The presence of glacial deposits and the proximity to active glacial formations suggest that a large amount of water was released into the area, causing the observed erosion patterns and the flooding.
Explanation:
consider the formula for average velocity. how is the average velocity of an object related to position?
The average velocity of an object is a measure of how its position changes over a given time interval. It relates the object's displacement (change in position) to the time taken for that displacement to occur.
The average velocity of an object is related to its position through the formula for average velocity, which is defined as the change in position divided by the change in time. Mathematically, average velocity (Vavg) is given by the formula:
Vavg = (Δx) / (Δt)
Where Δx represents the change in position and Δt represents the change in time.
The average velocity describes the displacement of an object over a given time interval. It indicates how far an object has moved and in what direction during that time period. The sign of the velocity (+ or -) indicates the direction of motion.
A positive average velocity indicates that the object is moving in the positive direction, while a negative average velocity indicates motion in the negative direction.
If an object's position remains constant, then there is no change in position (Δx = 0), and therefore the average velocity is zero. This implies that the object is at rest, not moving.
On the other hand, if an object undergoes a change in position, the average velocity will be non-zero. The magnitude of the average velocity represents the speed at which the object is moving, and the direction of the average velocity indicates the object's direction of motion.
For more such questions on average velocity visit;
https://brainly.com/question/29483294
#SPJ8
What is the energy of a photon with a frequency of 2.2 x 1016 Hz? Planck's constant is 6.63 x 10-34 Jos.
O 1.5 x 10-17
8.8 x 10-17
O 1.5 10-16
O 8.8 10-16
Answer: 1.5 × 10^-17
Explanation:
Given the following :
Frequency(f) = 2.2 × 10^16 Hz
Planck's constant(h) = 6.63 × 10^-34
The energy of a photon 'E' is given as the product of frequency and the planck's constant
E = hf
E = (6.63 × 10^-34) × (2.2 × 10^16)
E = 6.63 × 2.2 × 10^(-34 +16)
E = 14.586 × 10^-18
E = 1.4586 × 10^-17
E = 1.5 × 10-17 (2 S. F)
Answer:
C. 1.5 × 10–16 J
Explanation:
if a star was moved twice as far away, how bright would it appear?
If a star is moved twice as far away, it would appear one-fourth as bright.
The apparent brightness of a star depends on its distance and the inverse square law. According to the inverse square law, the brightness of an object decreases as the square of its distance increases. If a star is moved twice as far away, its distance from the observer would be doubled.
Applying the inverse square law, the brightness would decrease by a factor of four (2^2). This means the star would appear one-fourth as bright as before. The amount of light reaching the observer decreases as the distance increases, leading to a decrease in apparent brightness. Therefore, moving a star twice as far away reduces its apparent brightness to one-fourth of its original value.
To learn more about inverse square law
Click here brainly.com/question/32187932
#SPJ11
a helicopter cruises at a constant velocity when the total thrust of the engines is 5,000 N. How much air resistance acts on the jet ??
Hi there!
\(\large\boxed{F_{A} = -5000N}}\)
For an object to be moving at constant velocity:
∑F = 0 (The sum of the forces acting on the object MUST be 0 N)
We can do a summation of forces for this helicopter in the HORIZONTAL direction:
∑Fₓ = Ft (Force of Thrust) + Fa (Force of air resistance) = 0
Thus, if we substitute in the given value for Ft:
Ft = -Fa
5000N = -Fa
Fa = -5000N
20 points please help and explain thank you so much!!!A car traveling north at a velocity of 15m/s needs to pass a slower moving vehicle. The driver of the car speeds up to 20 meters per second to get around the slower vehicle. The passing time is 5.8 seconds. What is the car's acceleration?
The acceleration of the car when get around the slower vehicle is 0.86m/s².
The initial velocity U of the car is 15m/s.
The final velocity V of the car is 20m/s.
The time taken by the car to overtake the slow vehicle is 5.8 seconds.
To find the acceleration of the car, we can use the equation of motion,
The equation is,
V = U + at
Where,
V is final velocity of object,
U is the initial velocity if the object,
a is the acceleration of the body,
t is the time taken by the body to accelerate,
Putting all the values of find the acceleration a of the car,
V = U + at
20 = 15 + a5.8
5 = a5.8
a = 5/5.8
a = 0.86 m/s².
So, the acceleration of the car is 0.86 m/s
To know more about the acceleration, visit,
https://brainly.com/question/460763
#SPJ1
Which of the following is an example of physical weathering?
O calcium carbonate in limestone changes to calcium hydrogen carbonate
O flow water carves erosion channels in a hillside
Ostalactites precipitate in a cave
O acid rain corrodes a monument
Answer:
acid rain corrodes a monument. d.
Explanation:it only makes sense because its doing something to a physical object.and the other ones aren't.
The thermal energy transferred during a calorimetry experiment is equal to 32 J. This is equivalent to the work done by a 2-N force
A. acting over a distance of 16 m
B. acting over a distance of 64 m
C. acting for 16 s
D. acting on a 16-kg mass
A thermal energy of 32J is equivalent to the work done by a 2-N force acting over a distance of 16 m (option A).
What is work done?Work done is measure of energy expended in moving an object. It can be said that no work is done if the object does not move.
Work done can be calculated by multiplying the force by distance as follows:
Work done = force × distance
According to this question, the thermal energy transferred during a calorimetry experiment is equal to 32 J.
Since work done is force multiplied by distance, this suggests that a force of 2N will be acting over a distance of 16metres.
Learn more about work done at: https://brainly.com/question/13662169
#SPJ1
For Q6. high school student holds a backpack one meter above the ground. Which of the following free-body diagrams best represents this situation?
The image that shows the free body diagram of the bag pack is option J.
What is the free body diagram?The term free body diagram has to do with the way the that the forces that act on an object are able to carry on. We know that an object is always acted upon by a system of forces. The forces that act on the object could be balanced or unbalanced. If the system of forces is balanced than the object would remain stationary. If the system of forces is not balanced then the object would move.
In this case, we have been told that high school student holds a backpack one meter above the ground. We know that we expect that the free body diagram would be able to show us the forces that act on the bag and this would include the weight of the bag which is the gravitational pull on it and the tension on the rope that holds the bag.
Learn more about free body diagram:https://brainly.com/question/10148657
#SPJ1
Bob has been asked to produce a fuse that has 3. 2 ohms of resistance. He cannot change he thickness of the wire but can change its length. Explain in detail how bob could work out what length of wire to use. You will need to describe the experiment he will need to carry out , any hazards and any variables involved
The wire's resistance at different lengths and analyzing the data, Bob can determine the appropriate length of wire needed to achieve a resistance of 3.2 ohms.
To determine the length of wire Bob needs to achieve a resistance of 3.2 ohms, he can perform an experiment using the wire to measure its resistance at different lengths. Here's a step-by-step explanation of how Bob can carry out the experiment:
1. Gather materials: Bob will need the wire, a power supply (e.g., a battery), an ammeter (to measure current), and a voltmeter (to measure voltage). Ensure all equipment is properly calibrated and suitable for the current and voltage levels.
2. Design a circuit: Bob should set up a simple circuit consisting of the power supply connected in series with the wire, the ammeter to measure the current passing through the wire, and the voltmeter connected across the wire to measure the voltage drop.
3. Safety precautions: It is important for Bob to follow safety protocols while conducting the experiment. He should handle the wire and electrical equipment with care, avoid touching exposed wires, and ensure the circuit is properly insulated. Additionally, he should wear appropriate safety gear such as gloves and goggles.
4. Initial wire length: Bob should start with an initial length of wire and measure its resistance using a multimeter or an ohmmeter. This measurement will serve as the baseline value.
5. Adjusting wire length: Bob can then modify the length of the wire by cutting or extending it. For each length, he needs to ensure the wire is securely connected in the circuit.
6. Recording data: At each wire length, Bob should record the current (I) and voltage (V) values from the ammeter and voltmeter, respectively. These readings will help him calculate the resistance using Ohm's law: R = V/I.
7. Repeat measurements: Bob should repeat the measurements for several different wire lengths to gather enough data points to analyze and determine a trend.
8. Data analysis: Bob can plot a graph of wire length (x-axis) against resistance (y-axis) using the recorded data. By observing the relationship between wire length and resistance, he can identify the length of wire that corresponds to a resistance of 3.2 ohms.
Variables and Hazards:
Independent variable: Wire length. Bob can manipulate this variable by changing the wire's length.
Dependent variable: Resistance. Bob will measure this variable and use it to determine the relationship with the wire length.
Control variables: Bob should keep other factors constant throughout the experiment, such as the thickness of the wire and the material used.
Hazards: The main hazards involved in this experiment are electrical hazards, including electric shock and short circuits. Bob should ensure the circuit is properly insulated, handle the wires and equipment safely, and follow electrical safety guidelines.
To know more about resistance refer here
https://brainly.com/question/30799966#
#SPJ11
Scuba tanks should always have some pressure in them to prevent______
The given statement is incomplete and is missing the options or the possible answer from which we could determine the appropriate term that fits in the blank.
Therefore, I can provide a general answer to the question.
Scuba tanks should always have some pressure in them to prevent moisture contamination. When there is no pressure in the tanks, there is a chance that the moisture can enter the tank and contaminate the air inside. Therefore, it is crucial to have some pressure in the scuba tank. Also, a full scuba tank is around 3000 psi, and an empty scuba tank is around 200 psi. A scuba tank should be visually inspected annually and tested hydrostatically every five years to ensure that they are safe to use.
To know more about Scuba tanks, visit:
https://brainly.com/question/31578465
#SPJ11
Why are infrared waves ineffective for treating cancer
They do not transmit sufficient energy to kill cancer cells. Longer wavelengths than visible light define infrared waves (IR), a kind of electromagnetic radiation.
What are infrared ray?The electromagnetic radiation known as infrared, also referred to as infrared light, has wavelengths that are longer than those of visible light and shorter than those of radio waves. Wavelength range and sources: between 780 nm and 1 mm. Infrared radiation (IR), commonly referred to as heat radiation, is that region of the electromagnetic spectrum with wavelengths above red visible light, between 780 nm and 1 mm. IR can be divided into three groups: IR-A (780 nm-1.4 m), IR-B (1.4-3 m), and IR-C, commonly known as far-IR (3 m-1 mm).
IR wavelengths range from 700 nanometers (frequency 430 THz), which corresponds to the visible spectrum's notional red edge, to 1 millimeter (300 GHz). Long-term IR exposure, according to medical studies, can harm the lens, cornea, and retina, causing cataracts, corneal ulcers, and retinal burns, respectively. Workers can use gear with IR filters or reflective coating to help prevent long-term IR exposure. Electrical warmers, food-cooking appliances, remote controls, optical fibres, security systems, and thermal imaging cameras that can see individuals in the dark all employ infrared (IR) light. whereas, gamma rays, which have the highest energy (and thus the most penetrating), for instance, have higher frequencies than infrared waves.Gamma rays are the electromagnetic waves with the highest energy, highest frequency (300 EHz), and shortest wavelengths (1 pm) (1,24 MeV) and are used in treating cancer.
To know more about the infrared rays, visit:
https://brainly.com/question/17029597
#SPJ1
Southern California has been in drought conditions for several years which means it is much drier than normal. In the summer months, the temperature can easily reach 110 degrees Fahrenheit. You are investigating how plants respond to the dry, extremely hot conditions. It is approximately 1 pm when you make your observation. What would you expect to see? And why? To avoid losing too much water, the stomata will be closed. O To prevent photosynthesis from occurring, the stomata will be closed. The stomata will be open to allow the plant to absorb carbon dioxide. O The plant needs water so the stomata must stay open.
In the dry and extremely hot conditions of Southern California during summer, at approximately 1 pm, one would expect to see that the stomata of plants are closed. This is because to avoid excessive water loss through transpiration, plants tend to close their stomata, which are tiny openings on the surface of leaves responsible for gas exchange. Closing the stomata helps to conserve water by reducing the loss of water vapor from the plant's leaves.
Stomata play a crucial role in gas exchange during photosynthesis. However, when plants are exposed to hot and dry conditions, the closure of stomata becomes essential to prevent excessive water loss. By closing their stomata, plants can reduce transpiration and conserve water. Transpiration is the process through which water vapor escapes from the plant through the stomata.
Closing the stomata helps in minimizing water loss by reducing the evaporation of water from the plant's leaves into the surrounding dry air. This is particularly important in drought conditions when water availability is limited. By conserving water, plants can better withstand the dry and hot environment.
While the closure of stomata helps in water conservation, it also has an impact on photosynthesis. When the stomata are closed, the exchange of gases, including carbon dioxide (CO2) for photosynthesis, is restricted. As a result, the rate of photosynthesis decreases. However, in extremely hot and dry conditions, the priority for the plant is to conserve water rather than carry out photosynthesis. Therefore, it is expected that the stomata will be closed to limit water loss, even though this reduces the availability of CO2 for photosynthesis.
To learn more about Water click here:
brainly.com/question/29387296
#SPJ11
How much water will flow in 30 secs through 200 mm of capillary tube of 1.50 mm in diameter, if the pressure difference across the tube is 6660N/m² and viscosity is 8.01x10 kg/ms
The water outflow in 30 secs through 200 mm of the capillary tube is mathematically given as
\(Qo=1.6 \times 10^{2} \mathrm{~mL}\)
What is the water outflow in 30 secs through 200 mm of the capillary tube?\(\begin{aligned}\Delta P &=6660 \mathrm{~m} / \mathrm{m}^{2} \\\mu &=8.01 \times 10^{-4} \text { Pas } \\t &=30 \mathrm{~s} \\L &=200 \mathrm{~mm}=200 \times 10^{-3} \mathrm{~m} \\D &=1.5 \mathrm{~mm}=1.5 \times 10^{-3} \mathrm{~m} \Rightarrow \gamma=\frac{1.5 \times 10^{-3}}{2} \mathrm{~m}\end{aligned}\)
Generally, the equation for Rate of flow of Liquid is mathematically given as
\(\\$$Q=\frac{\pi r^{4} \times \Delta P}{8 \mu L}\)
$$
Where dP is pressure difference r is the radius
\(\mu\) is the viscosity of water
L is the length of the pipe
\(Q=\frac{\pi \times\left(\frac{1.5 \times 10^{-3}}{2}\right)^{4} \times 6660}{8 \times 8.01 \times 10^{-4} \times 200 \times 10^{-3}}\)
\(Q=5.2 \mathrm{~mL} / \mathrm{s}\)
In $30s the quantity that flows out of the tube
\(&Qo=5.2 \times 30 \\&Qo=1.6 \times 10^{2} \mathrm{~mL}\)
In conclusion, the quantity that flows out of the tube
\(Qo=1.6 \times 10^{2} \mathrm{~mL}\)
Read more about the flows rate
https://brainly.com/question/27880305
#SPJ1